Logical and Critical Thinking-II-1
Logical and Critical Thinking-II-1
Dear Students,
We are delighted to welcome you to this exciting and transformative journey into the world of logical
and critical thinking. This module is designed to sharpen your analytical abilities, enhance your
decision-making skills, and empower you to approach problems with a clear and structured mindset.
The modules that we will cover in this course are outlined as follows:
Module Topics
Data arrangements and Blood relations – Advanced, Cryptarithmetic, Creative Thinking
1
and Inductive Reasoning, Rebuz Puzzles
Data Sufficiency, Coding & decoding, Series, Analogy, Odd man out and Visual reasoning
3 Advanced, Critical Reasoning(Premise, Assumption, Conclusion, Mimic the pattern),
Guesstimate
Please note that, our training process includes periodic assessments (Internal and External
Assessments), that will be crucial parts of your learning journey. We encourage you to approach these
assessments with enthusiasm and a positive mindset.
In addition, the assessment rubrics given below will be integral to your overall grading:
Number
Category of Marks Pattern Syllabus
Instances
Module 1- Assessment 1
Internal Number of Questions – 30
3 * 10 30 Module 2- Assessment 2
Assessment Duration - 45 mins
Module 3- Assessment 3
Attendance 1 5 75% -
Coursera -
1 5 -
Certificate
Learning Course 1 10 - -
External Number of Questions – 50 All modules
1 50
Assessment Duration - 75 mins
Total 100 -
We encourage you to actively participate, ask questions, and collaborate with your peers to maximize
your learning experience.
For any questions or clarifications, feel free to visit our expert trainers at Room CLC06 (Library Block -
Reading Hall) between 9:00 AM and 4:30 PM, Monday through Friday. Alternatively, you can reach out
to us via email at [email protected].
We are thrilled to have you with us and look forward to seeing you thrive in these training sessions.
Let’s embark on this journey together, and make the most of every opportunity to grow and excel.
Concept 1: Linear arrangements: Linear Left ____ Manish _____ Ramesh right
arrangement is the arrangement of given items
with the help of the given clues. The main idea Manish Raju Ramesh
here is to represent the given data in an
organized manner. Once that is done, solving Manish Raju Ramesh Shekar
individual problems becomes very easy
2. Who would be second from the left?
Concept 2: Circular arrangements: Questions
A. Manish
on circular arrangements involve arrangement
B. Raju
of people sitting around a table or
C. Ramesh
arrangement of things in a circular manner.
D. Shekar
In case of people sitting around a table, the
Answer: B
table could be of any shape– rectangular,
square, circular, etc. Circular arrangement is Solution:
different from linear arrangement in that there
is no first or last position and hence there is no Left ____ Manish _____ Ramesh right
reference point. Therefore, the positions taken Manish Raju Ramesh
by the objects are only relative to each other. Manish Raju Ramesh Shekar
Once you read the question, first draw the Directions for questions 3 to 7: Six athletes
shape specified in the question and then draw Anu, Benny, Chandru, Dolly, Esha, Furpreet
the slots for the arrangement. Then solve the compete in a race. The outcomes are as follows
given clues one by one. :
Solution:
From clue 2:
__ D B __ E A From clue 1:
__ __ B __ __ __
From clue 5:
F D B __ E A From clue 4:
__ __ B __ E __
Left over person is C.
F D B C E AAs it can be clearly seen from the From clue 3:
layout, one person separate Furpreet and __ __ B __ E A
Benny (Dolly).
From clue 2:
5. Who secured the middle position in the
__ D B __ E A
race?
Solution:
From clue 1:
__ __ B __ __ __
From clue 4:
__ __ B __ E __
From clue 3:
__ __ B __ E A
There are 3 people between Farhan and
From clue 2:
Emmy.
__ D B __ E A
9. Who is to the immediate left of Chandani?
From clue 5:
A. Annu
F D B __ E A
B. Bansi
C. Farhan
Left over person is C.
D. Cannot be determined
FDBCEA
Answer: A
Since Chandru secured the 4th position, we can
conclude that 3 swimmers finished ahead of Solution:
him.
3. R is adjacent to M. 1. -- -- -- -- E -- -- -- -- 1 234 5 67 89
11. 9 people, A to I are seated in a row to be C is the sixth person from the left end in the
photographed. Their seating arrangement is in photograph
accordance with the following conditions: Directions for questions 12 to 15:
E sits at the centre. Six friends Priya, Qutin, Rama, Sachin, Tilak and
F and I sit on the corner seats. Umesh are sitting around the hexagonal table
each at one corner and are facing the centre of
H is equidistant from both I and E. the hexagonal. Priya is second to the left of
D sits three places to the right of E. Umesh. Qutin is neighbour of Rama and
Sachin. Tilak is second to the left of Sachin.
A is two places to the left of D and next to H
12. Who is sitting opposite to Qutin ?
G is in between B and C and C is closer to E.
A. Rama
Who is the sixth person from the left end in the B. Tilak
photograph? C. Umesh
D. Sachin
A. A
B. B Answer: B
C. C
Solution: 14. Which of the following are the neighbours
of Tilak ?
Answer: A
Solution:
A. Priya
B. Umesh
C. Umesh
D. Rama
E. Data Inadequate
Answer: A
15. Which one is sitting opposite to Tilak ?
Solution:
A. Rama
B. Qutin
C. Sachin
D. Cannot be determined
Answer: B
Solution:
17. Which of the following is not a correct
combination?
A. C tasteless
Directions for Questions 16 to 20:
B. F sweet
There are six fruits A to F, among which two are C. D sour
sweet in taste, two are sour in taste and two D. E sour
are tasteless. They are arranged in a basket
Answer: D
circularly.
Solution:
1) A is adjacent to F.
A. A
B. B
C. C
D. D 18. Which dishes are adjacent to D?
Answer: D A. A and C
B. E and C
Solution: C. D and F
D. A and E
Answer: B
Solution:
19. Which of the following is true? Directions for Questions 21 to 22: There are
five different buildings.
A. One of the tasteless fruit is opposite to
sour fruit Following are the clues given:
B. A sour fruit is opposite to a sweet fruit
1) A to E in a row.
C. A is a sweet fruit
D. More than one of the options 2) A is to the right of B.
Answer: D 3) E is to the left of C and right of A.
Solution: 4) B is to the right of D.
A. A
B. C
C. D
D. E
Answer: B
Solution:
A is to the right of B.
B A
20. Which of the following fruits can be
together? B is to the right of D.
D B A
A. B and E
B. D and F
E is to the left of C and right of A.
C. B and C
DBAEC
D. A and B
22. How many buildings are there between D
Answer: A
and C?
Solution:
A. 1
B. 2
C. 3
D. 0
Answer: C
Solution:
2) Seema is not the neighbour of Priya. 25. Which of the following is true ?
3) Vijay is the neighbour of Upasna. A. Priya is to the immediate right of
4) Qureshi is not between Seema and Qureshi.
Wahadutt. B. Ravi is between Upasna and Vijay.
C. Qureshi is to the immediate left of
5) Wahadutt is not between Upasna and Wahadutt.
Seema. D. Upasna is between Wahadutt and
23. Which of the following options represents Seema.
neighbours ? Answer: C
A. Ravi & Vijay Solution:
B. Upasna & Tina
C. Ravi & Tina In the 1st possible arrangement, Qureshi is to
D. Upasna & Wahadutt the immediate left of Wahadutt.
Answer: C
Solution:
So the order is : N, O, P, Q, R, S, T.
Directions for questions 28 to 30: Study the
following information to answer the given 30. Who is standing between Om and Quiti?
questions:
A. Pooja
In a March Past, seven soldiers are standing in B. Rishit
a row. Quiti is standing left to Rishit but right C. Nisha
to Pooja. Om is standing right to Nisha and left D. None of the options
to Pooja. Similarly, Shashi is standing right to
Answer: A
Rishit and left to Tanya.
Solution:
28. Who is standing in the middle?
Q is left to R and to the right of P
i.e. P, Q, R.
So the order is : N, O, P, Q, R, S, T.
*****
Blood Relations Mother’s husband is my Father;
My Father’s mother is my Grandmother;
Concept My Grandmother’s only son is my Father.
Blood relation is one of the most often asked 2. Anup introduces Rashmi as the daughter of
logical reasoning topics. A set of clues are given the only sister of his father’s wife. How is
to determine the blood relationship between Rashmi related to Anup?
individuals in questions. Similar to data
arrangement, once the given set of data is A. Cousin
represented in an organized manner, these B. Son
questions can be cracked in almost no time. C. Uncle
The following symbols help in representing the D. Son-in-law
given clues so as to solve the problems easily. Answer: A
Solution:
A. Mother
B. Father
C. Maternal uncle
D. None of these
Answer: D
Solution:
Answer: A
My maternal uncle’s father is my grandfather. My father’s brother is my uncle.
Grandfather's only daughter is my mother and So the man is Gowtham’s uncle.
I am her son. Raj is Priya’s brother.
8. Pointing to Gokul, Meera says, "I am the
daughter of the only son of his grandfather."
How Meera is related to Gokul?
5. A’s son B is married with C whose brother D
is married to E the sister of B. How D is related A. Niece or aunt
to A? B. Daughter or mother
C. Sister or cousin
A. Brother
D. Cannot be determined
B. Son-in-law
C. Brother-in-law Answer: C
D. Cousin
Solution:
Answer: B
Meera is the daughter of the only son of
Solution: Gokul’s grandfather, both maternal and
paternal. Hence, it’s clear that Meera can be
Since E is the sister of B
the sister or cousin of Gokul.
Therefore, A is the father of E
but D is the husband of E. 9. Showing the woman receiving the prize
Hence, D is the son-in-law of A. Sania said, “She is the sister of my uncle’s son”.
What is the woman to Sania?
6. Introducing a boy, Santhosh said, “His
mother is the only daughter of my mother-in- A. Son
law”. How is Santhosh related to the boy? B. Brother-in-law
C. Nephew
A. Father
D. Cousin
B. Brother
C. Uncle Answer: D
D. Husband
Solution:
Answer: A
Sister of my uncle’s son is my uncle’s daughter,
Only daughter of mother-in-law -> wife i.e. who is my cousin. i.e. The women is Sania’s
boy’s mother is Santhosh’s wife. Santhosh is cousin.
the father of the boy.
10. Prakash is the son of Pramod. Neha is the
7. Pointing to a man on the platform Gowtham daughter of Abhishek. Ruchi is the mother of
said, “He is the brother of the father of my Neha. Awadhesh is the brother of Neha. How
mother’s son”. What is the man to Gowtham? is Abhishek related to Awadhesh?
A. Uncle A. Father
B. Father B. Brother
C. Brother C. Son
D. Nephew D. Grandfather
Answer: A Answer: A
Solution: Solution:
Answer: D
Solution:
*****
A.
Cryptarithmetic 17
B. 18
Cryptarithmetic problems are puzzles where
letters stand in for digits in mathematical
equations. The goal is to find the digit each C. 19
letter represents while maintaining the D. 20
integrity of the equation.
Answer A.
Basic Concepts
Explanation: - YOUR + YOU = HEART
1. Each letter represents a unique digit
Here they have given O as 4. Y can take value
(0-9). as 9, the maximum value of carryover is 1,
2. No two different letters can represent where 1 + 9 = 10, where we will get 2 digit
the same digit. number as Y value is 9 and O value is 4 (9+4 =
13), Here A is 3 and we are getting carryover
3. The first letter of a number can't be is “1”. Y value is 9 and carryover gives us 10
zero. means E is 0 and H is 1. So, now we have
found all the numbers except URT try to
Practice Questions eliminate the number which is used already
(1, 3, 4, 9, 10). Now we are left out with (2, 5,
1. COCA + COTA = OASIS each letter consists
6, 7, 8).
unique digit (0-9) then find S+O+T+I+C+A
Here O is 4, U can take value of 5, 6, 7 (or) 8.
A. 28 Value of R should be greater than 4. Let us say
B. 27 value of U is 2, (U+O) = (2+4) = 6 which means
C. 26 R value is 6. (R + U) = (6+2) = 8.
D. 25
Y+U+R+E = 9 + 2 + 6 + 0 = 17.
Answer C.
3. LET+LEE=ALL THEN A+L+L =? (Assume E=5)
Explanation: - COCA + COTA = OASIS
A. 1
Addition of two single digit numbers is giving B. 3
two digit numbers. So value of C should be 5 C. 5
or greater than that. So, carryover is 1 and D. 7
value of O is 1. So, value of S is 2.
Answer C.
Addition of two same numbers should have to
be 2. Here A cannot be 1 because already Explanation: - L E T + L E E = A L L
value of O is 1. A cannot be 1 because already Here they have given E value as 5. (E + E) = (5
value of O is 1. A cannot be (0, 3, 4, 5) .So, +5) = 10. L value can never be 0, so we should
value of A is 6 (6 + 6 = 12) and we will have have carry over. So, T should take value
carry over 1. greater than 4. If value of T is 6 (T + E) = (6 + 5)
Value of C is 3. But we didn’t have carry over = 11 and value of L is 1, there will be carryover
value of C should be more than that we have of 1.
used (1, 2, 6). C value can be 8 we will have (E + E) = (5 + 5) = 10 and carryover 1 => (10 +
carry over as 1. T should be 0. ‘ 1) = 11 Where L is ‘1’ (L+ L) = (1 + 1+ carryover
S + O + T + I + C + A => 2 + 1 + 0 + 9 + 8 + 6 = (1)) -> A = 3
26. A + L + L = 3 + 1 + 1 = 5.
2. YOUR+YOU=HEART (O=4) find the value of 4. HERE=COMES-SHE, (assume S = 8) Find the
Y+U+R+E? value of R+H+O.
A. 14 Answer D.
B. 13
C. 12 Explanation: - Clearly, P = 1, U = 9, E = 0. As P
D. 11 will have gotten 1 carry from previous step, 1
is the max carry that can be generated. U +
Answer A. nothing = E, it should have been U itself; this
Explanation: - Let us rewrite it as HERE + SHE = means there is carry of 1 from previous step.
COMES. Here value of s is 8, value of E should U + 1 = E and 1 carry is generated to next step.
be 4 where (E+E) = (4+4) = S(8). So here (E + S) So, U + 1 > 10, only possible way to have this
= (8 + 4 + carryover (1)) = 13, where m value is is when U is 9. E = U + 1 = 9 + 1 = 10 => 0(1
3 there will be carryover is 1. H will be carry to next).
maximum value 9 + 1 = 10 where O value is 0
and C is 1. A + R = 0. This is possible if, A = 5, R = 5, but,
both can’t take same values. So its possible
R + H +O = 5 + 9 +0 = 14. with (8,2), (7,3), (6,4), (4,6), (3,7), (2,8)/ Let’s
5. If POINT + ZERO = ENERGY, then E + N + E + do trial and error take (2,8) and replace the
R + G + Y =? new values.
YEARS A. 13
B. 14
----------
C. 15
What is the value for Y+E+A+R+S? D. 16
A. 13 Answer A
B. 16
Explanation: -
C. 18
D. 20 Arrange A-J(0-9) likewise K-L(0-9) and U-Z(0-5)
the code is S=8,H=7,E=4 so 8+7+4=19 or
Answer A
(HE)^H must be in the range 21 to 29 so that
Explanation: - A F T E R result must be less than 999 (3 digit ,as SHE)
.so as per rule sol is (25)^2=625. So
TWO S+H+E=6+2+5=13
--------------------- 14. GO+TO=OUT, O+U+T=?
YEARS A. 5
Here we have maximum values; F will be 9 B. 3
with carryover 1 so F + carryover = 10 means E C. 2
is 0. D. 4
A. 123
18. COMES – SHE = HERE. Find the value of Therefore possible values of B and O are 2 and
(S/E)*R. 3 respectively.
A. 30 R = 1, B = 2, O = 3, A = 4, U = 5, G = 6, V = 7, E
B. 10 = 8, N = 9
C. 40
So, O + R + A + N + G + E = 3 + 1 + 4 + 9 + 6 +8
D. 45
= 31.
Answer: Option B.
20. EUROPA – VENUS = URANUS. Find the
Explanation: - H should have maximum value value of U+R+A+N+U+S.
9, as a result C should be 1 and O should be 0.
A. 20
Let us consider E as 4 (E + E) = 4 and S = 8. (E + B. 25
S) = (4 + 8) = 12, where as M is 2 with C. 30
carryover 1. D. 35
Now sum of (R + H) should give us E which is Answer: Option B.
4. H value is 5 + 9 = 14 (R + H) = 14 will have
carryover 1 here. So, M is going to be 3. Explanation: - Now, let’s start with S. S value
as 1, (S + S) = 1 + 1 = 2. Now, A becomes 2.
COMES = 10348
Let us assume U as 5 (U + U) = (5 + 5) = 10, P
(S/E) * R = (8/4) * 5 = 2 * 5 = 10. becomes 0 with carryover 1. Let us consider N
as 4. So, (4 + 4 + carryover (1)) = (N + N +
19. BANANA + GUAVA = ORANGE. Find the carryover (1)) = 9. So O becomes 9.
value of O+R+A+N+G+E.
Now, we have found the value of U to be S, A
A. 20 + E = R and R + V = U (5)
B. 25
C. 23 Let us consider, E as 6 then R will be 8.
D. 31
If R = 8, V will be 7 (R + V) = (8 + 7) = 15. So U
Answer: Option D. will be 5 with carryover 1.
Answer: Option B. A. 12
B. 16
Explanation: - C. 24
apple+litchi=cherry D. 28
31176+794829=826005
c+h+e+r+r+y=8+2+6+0+0+5=21 Answer: Option A.
The values: O=9, T=8, R =7, E=5, X=4, I=1, and --------------------
the left out numbers are 6, 3, 2. We know F +
1 = S i.e. F and S will take 2 and 3, and at last, DCBA
the Y will take 6.
Outcome is a 4 digit number, i.e., no carry
The value of S * I * X = 3 * 1 * 4 = 12. from A when we multiply this by 4. So,
possible value of A = 1 or 2
26. SEND + MORE = MONEY. Find the value
for MONEY Now A can’t be 1, because in final result A is in
Unit place and 1 is not possible when we
A. 10652 multiply any number by So, A = 2 Now we
B. 15042 have
C. 12564
D. 13462 2BCD
*4
Answer: Option A.
---------------
Explanation: -
DBC2
SEND
Now you can see that D = 8,
+MORE
2BC8
-----------------
*4
MONEY
--------------------
-----------------
8CB2
From the given data, the value of M will be
1 because it is the only carry-over possible
Now you can see that no carry must be there
from the sum of 2 single digit numbers in
when we multiply B with 4, so possible values
column 4. M = 1, S + 1 = a (two digit number).
of B is 1 or 2
So S = 1 and O cannot be 1 but 0. Also E and N
are consecutive. With trial and error Let us consider B = 1, i.e.,
possibilities, we get SEND = 9567, MORE =
1085 and MONEY = 10652. 21C8
*4 Answer: Option D.
----------- Explanation: -
8C12 T O M
28. RAT * T = CAT. Find the value of √(CAT). O + A = A, from this we can tell that O = 0. T +
N = O (O should be a two digit number ending
A. 21 in zero, only then G will be 1). Sum of T and N
B. 23 should be 10 i.e. T (6) + N (4) = 10. M + G = T,
C. 25 from this we will get the value of M as 1.
D. 27
O=0, G=1, N=4, M=5, T=6 and the left out
Answer: Option C numbers are 2, 3, 7, 8, 9, from this A can take
any value. There is no definite value for A.
Explanation: - RAT * T = CAT
15. WORLD+TRADE=CENTER value of
All these numbers are square root because C+E+N+T+E+R =?
here we need to calculate √CAT
A. 26
So, the numbers are 441, 625 like this B. 12
C. 26
Where 32 ends in 9, 72 ends in 9 and 12 ends in D. 27
1. So, these numbers are not possible.
Answer: Option D.
Whereas 5 * 5 = 25, T * T = T.
Explanation: -
Considering CAT value to be 625 (25*5 = 625).
So R value should be 1 world=5 3 6 8 4
trade=7 6 0 4 2
So, √625 = 25. center=1 2 9 7 2 6=27
All these three CAT should have unique *****
number.
A. 9
B. 10
C. 13
D. Cannot be determined
Creative Thinking and Inductive Reasoning
Solving puzzles often requires a mix of logical thinking, pattern recognition, and sometimes a bit of
creativity.
Basic Concepts:
• Pattern Recognition: Identify repeating sequences or consistent changes.
• Logical Deduction: Use clues to eliminate options and find the answer.
• Working Backwards: Start from the solution and trace steps back.
• Breaking the Problem Down: Divide the puzzle into smaller, manageable parts.
Practice Questions
1. At McDonalds you can order Chicken McNuggets in boxes of 6, 9, and 20. What is the largest
number such that you can not order any combination of the above to achieve exactly the number
you want?
A. 41
B. 42
C. 43
Answer: C
Solution:
# of # of
Possible Combinations Possible Combinations
McNuggetts McNuggetts
1 None 16 None
2 None 17 None
3 None 18 9,9 or 6,6,6
4 None 19 None
5 None 20 20
6 6 21 6,6,9
7 None 22 None
8 None 23 None
9 9 24 6,9,9 or 6,6,6,6
10 None 25 None
11 None 26 6,20
12 6,6 27 9,9,9
13 None 28 None
14 None 29 9,20
15 6,9 30 6,6,9,9 or 6,6,6,6,6
# of # of
Possible Combinations Possible Combinations
McNuggetts McNuggetts
31 None 46 6,20,20
32 6,6,20 47 9,9,9,20
33 6,9,9,9 48 6,6,9,9,9,9
34 None 49 9,20,20
35 6,9,20 50 6,6,9,9,20
36 9,9,9,9 or 6,6,6,6,6,6 51 6,9,9,9,9,9
37 None 52 6,6,20,20
38 9,9,20 53 6,9,9,9,20
39 6,6,9,9,9
40 20,20
41 6,6,9,20
42 6,9,9,9,9
43 None
44 6,9,9,20
45 9,9,9,9,9
The data in the above tables are a list of the number of McNuggetts one would like to purchase
and then the combination or combinations of 6, 9 and/or 20 McNuggetts packages one could use
to purchase that number. If the number of McNuggetts cannot be purchased by only using a 6, 9,
or 20 pack of McNuggetts, then in the possible combinations column the word 'none' appears.
What we were trying to determine was what would be the largest quantity of McNuggetts one
could not purchase by combining only 6, 9 and/or 20 packs of McNuggetts?
From these tables, one can see that 43 McNuggetts is the largest number of McNuggetts that
cannot be purchased using only a 6, 9 or 20 pack of McNuggetts.
2. Three men walk into a motel and ask for a room. The desk clerk says a room is $30 so each man
pays $10 towards the cost. Later, the clerk realizes he made a mistake, that the room should have
been $25. He calls the bell boy over and asks him to refund the other $5 to the three men. The
bellboy, not wanting to mess with a lot of change dividing the $5 three ways, decides to lie about
the price, refunding each man $1 and keeping the other $2 for himself. Ultimately each man paid
$9 towards the room and the bellboy got $2, totaling $29. But the original charge was $30, where
did the extra $1 go?
A. 30$
B. 40$
C. 50$
Answer: A
Solution:
First Person Second Person Third Person
3. Joe's happiness is proportional to the function w2 *c (w squared * c), where w stands for daily
consumption of glasses of wine, and c stands of daily consumption of number of cigars. Wine
costs $3 a glass and cigars cost $2 each. His daily budget for both is $100. How should he divide
his money between wine and cigars to maximize his happiness? Assume he does not have to buy
integer numbers of either cigars or wine. Function of happiness is w2 * c, So both the values must
be an integer so that only we can get a integer value. So joe has to buy both wine and cigars.
Lets assume joe has 10$, how much wine and cigars he can buy for this money ,
Wine cigar
i) 3 glasses 0
iv) 0 glass 5 cigars. There are 4 cases, among these case i & iv are not possible,
because he will get the happiness value as 0. So take ii,
Happiness = 22 * 2 = 8
Take iii,
Happiness = 12 * 3 = 3.
So to get more happiness glasses of wine should be maximum and cigars should minimum.
He has 100$ with him, one glass of wine costs 3$, so he can buy 33 glasses for 99$, but he should
buy at least 1 cigars to get the maximum happiness. So reduce the glasses of wine to 32 glasses
for 96$. Remaining 4$, for this amount he can buy 2 cigars.
So he has to divide his money as 96$ for wine and 4$ for cigars.
4. A box contains two coins. One coin is heads on both sides and the other is heads on one side
and tails on the other. One coin is selected from the box at random and the face of one side is
observed. If the face is heads what is the probability that the other side is heads?
A. ½
B. 1/3
C. ¼
D. 1/5
Answer: A
Solution:
Coin 1:
Heads and Heads
Coin 2: Heads and Tails
We need a coin which has heads on both the faces (Coin 1). Probability
of selecting Coin 1 from the box with two coins= ½
5. You are a cook in a remote area with no clocks or other way of keeping time other than a 4 minute
hourglass and a 7 minute hourglass. You do have a stove however with water in a pot already boiling.
Somebody asks you for a 9 minute egg, and you know this person is a perfectionist and will be able to
tell if you undercook or overcook the eggs by even a few seconds. What is the least amount of turns
it will take to prepare the egg?
A. 4
B. 5
C. 6
D. 7
Answer: A
Solution:
i. Flip both hourglasses over and drop the egg into the water.
ii. When the 4-minute timer runs out, flip it over (4 minutes elapsed, 3 remaining on the 7-
minute timer).
iii. When the 7-minute timer runs out, flip it over. (7 minutes elapsed, 1 remaining in the 4-
minute timer)
iv. When the 4-minute timer runs out, flip the 7-minute timer over. (8 minutes elapsed. 6
minutes remained in the 7-minute timer, but flipping it over leaves one minute’s worth of
sand on top. When it runs out exactly nine minutes will have elapsed.)
Answer, 4 flips
A. 2, 2 and 8.
B. 2, 4 and 9.
C. 2, 2 and 9.
D. 2, 2 and 5.
AnsweR: C
Solution
Like all numbers, 36 can be broken down into primes in only one way: 36 = 2*2*3*3
From this breakdown we can discover the eight possible configurations of the children's ages:
1,1,36
1,2,18
1,3,12
1,4,9
1,6,6
2,2,9
2,3,6
3,3,4
If we look at the sums of the different configurations, we will find:
1+1+36=38
1+2+18=21
1+3+12=16
1+4+9=14
1+6+6=13
2+2+9=13
2+3+6=11
3+3+4=10
Since we know that person y couldn't tell what the children's ages were from the second hint
(and since person y no doubt knows his own house number) we can conclude that only two
configurations are possible - 1,6,6 and 2,2,9.
From the third clue, we know that there is an oldest child, so the configuration 1,6,6 is
impossible.
Therefore, the children's ages are 2, 2 and 9.
7. A woman is chosen at random among all women that have two children. She is asked do
you have at least one boy, and she answers 'yes.' What is the probability her other child is
a boy? Assume every pregnancy has a 50/50 chance to be a boy or a girl.
A. ½
B. 3/2
C. ¼
D. 5/6
Answer: A
Solution:
A woman is chosen who has 2 children. She has one boy, next child can be a boy or a girl, have
2 possibilities. Here no of possible events are 2,
No of favorable events is 1 (question is the probability that the next child is a boy).
So probability for the child to be a boy is = ½. (No. of fav. events/no. of possible events)
*****
Rebus Puzzle
Rebus puzzles are a unique form of word and picture puzzles that use images, letters, and symbols to
represent words or phrases. These visual conundrums challenge your ability to decipher the
underlying message hidden within the puzzle. The term “rebus” originates from the Latin phrase “non-
verbis sed rebus,” which means “not by words, but by things.” Essentially, a rebus puzzle
communicates its meaning through symbols rather than straightforward language.
1. Observe Carefully: The key to solving rebus puzzles lies in keen observation. Analyse each
element of the puzzle, paying attention to colours, shapes, and positions of images or letters.
2. Break it Down: Sometimes, rebus puzzles with answers can be complex at first glance. Break
down the puzzle into smaller parts and try to decipher the meaning of each element
individually.
3. Word Association: Associate the images or letters with common words or phrases that they
sound or look like. This approach can help you connect the dots and solve the puzzle.
4. Context Matters: Consider the context of the rebus puzzle’s meaning. Sometimes, a theme or
topic may provide clues to the hidden message.
Practice Questions
Directions:
A REBUS is a picture representation of a name, work, or phrase. Each "rebus" puzzle box below portrays
a common word or phrase. Can you guess what it is?
1.
A. Karate Kid
B. Two people in a car
C. None of the above
Answer: A
2.
A. Two leaves
B. To infinity and byond
C. None of the above
Answer: B
3.
A. Do not Disturb
B. End up behind the bars
C. None of the above
Answer: B
Explanation: Stand is written in upward direction “END UP”, and it is written behind bars. So, it is “END
Up behind bars”.
4.
Answer: B
Explanation:
Here in this image they have given Pen and Pin remove P from it and it is “Ball is in your Court”.
5.
A. Run Lee
B. Need for fast
C. None of the above
Answer: B
Solution:
From Needle we have to remove “le” and someone is running fast. So, basically it is “Need for fast”.
6.
A. Number Whelming
B. Over Whelming
C. None
Answer: B
Solution:
In this image they have given Odd numbers (1, 3, 5, 7, 9) and separating Whelming with line. So, it
is “Odds are overwhelming”.
7.
A. Rain in B
B. Scatter Brain
C. None
Answer: B
Explanation:
In the given image letters are scattered here and there it is “Scatter Brain”.
8.
A. Reasoning
B. Google G
C. None
Answer: A
Explanation:
9.
Answer: B
Solution:
Here END is written in upward direction and is written behind bars it is “END Up behind Bars”.
10.
A. I am watching you
B. See the forect for the trees
C. None
Answer: B
Solution:
Eyes, see for west. There are est. and for the trees. “See the forest for the trees”.
*****
A. 2° 30 * 10 – 11(25)/2
B. 4° 300 – 137.5 = 162.5
C. 8
D. 8° Reflex Angle = 360 – 162.5 = 197.5
E. 12 °
5. At what time between 5 and 6 are the hands
Answer: B 8-minute spaces apart?
Solution:
= 10 / 15 * 60 = 40 hours
*****
Calendars
Basic Concepts 2008 8th November to 8th January – 2 months
– 30+31 = 5 odd days
• An ordinary year has 1 one odd day, whereas
a leap year has 2 odd days. Saturday + 5 days = Thursday
• Century years should be divisible by 400 as a 3. Today is Monday . After 61 days , it will be :
check for leap year.
A. Tuesday
• For 100 years, there are 5 odd days; for 200 B. Thursday
years, there are 3 odd days; for 300 years, C. Saturday
there is only one odd day; for 400 years and D. Sunday
multiples of 400 years, there are 0 odd days.
Answer: C
Practice Questions
Solution:
1. On January 1, 2006, it was Sunday. What will
61/7 = 5 odd days
be the day of the week on January 1, 2010?
Sunday + 5 days = Saturday
A. Saturday
B. Sunday 4. The Last day of a century cannot be
C. Friday
A. Tuesday
D. Monday
B. Monday
Answer: C C. Wednesday
D. Friday
Solution:
Answer: A
2007 - Sunday + 1 day = Monday
Solution:
2008- Monday + 1 day = Tuesday
100 years contains 5 odd days. So the last day
2009 -Tuesday + 2 day = Thursday
is Friday.
2010 - Thursday + 1 day = Friday
200 years contains 3 odd days. So the last day
th
2. On 8 November 2006, Wednesday falls. is Wednesday.
Find out what day was the week on 8th January
300 years contains 1 odd days. So the last day
2009
is Monday.
A. Saturday
400 years contains 0 odd days. So the last day
B. Thursday
is Sunday.
C. Friday
D. Monday So it can be Tuesday, Thursday or Saturday.
The Only option is A – Tuesday.
Answer: B
5. What was the day of the week on 16th July,
Solution:
1776?
2006 – Wednesday
A. Tuesday
2007 – Wednesday + 1 = Thursday B. Monday
C. Wednesday
2008 (Leap Year)- Thursday +2 = Saturday D. Friday
Answer: A
Solution: B. Monday
C. Wednesday
Shortcut:
D. Friday
1776 = 1600+100+76 = 0+ 5 +76+19 = 100/7 –
Answer: A
2 odd days
Solution:
July Month code – 5
Feb 1 – mar 3 – 29 days + 2 = 31/ 7 = 3
Day – 16/7 – 2
Wednesday + 3 days - Saturday
Total - 2+ 5 +2 = 9 /7 = 2 - Tuesday
9. Rohit's birthday was celebrated on Monday,
6. If 25th December 1990 is a Tuesday, what day
16th January 1991. When would be celebrated
is the 25th December 2020?
his birthday again on Monday.
A. Tuesday
A. 1997
B. Monday
B. 2005
C. Wednesday
C. 2000
D. Friday
D. 2002
Answer: D
Answer: D
Solution:
Solution:
Shortcut:
Leap Year Shortcut:
First we need to calculate the odd days from
Leap year – 28
1990 to 2020, if we do that then we get the odd
number of days as Leap year +1 - 6
1+1+2+1+1+1+2+1+1+1+2+1+1+1+2+1+1+1+2
Leap Year +2 – 11
+1+1+1+2+1+1+1+2+1+1+1+1 = 38 or 3 odd
days. , Leap Year + 3 -11
So the answer will be 3 odd days from 10. How many times does 29 February come in
Tuesday, which is Friday. 400 years?
7. On 8th march 2005, Wednesday falls what A. 97
day of the week was it on 8th March, 2004 ? B. 99
C. 100
A. Tuesday
D. 98
B. Monday
C. Wednesday Answer: A
D. Friday
Solution:
Answer: B
In 100 years - 24 leap years
Solution:
In 400 years – 24*4 – 96
8th March 2005 is Wednesday. So, 8th March
2004 is Tuesday. 400th year is a leap year
Watch, Time, Count down, Rocket, Satellite Plant, Cotton, Yarn, Cloth, Saree
Logical Sequence:
Logical Sequence:
Logical Sequence:
Logical Sequence:
Logical Sequence:
• Weak Argument: Irrelevant to the Clearly, for better economic gain, losses should
statement, based on assumptions, or has be reduced and income increased. So, only
personal opinions. course I follows.
Since its launching in 1881, Vayudoot has so far E. Both I and II follow
accumulated losses amounting to Rs 153 crore.
Answer: E
Courses of Action:
Solution:
I. Vayudoot should be directed to reduce
The situation demands creating awareness
wasteful expenditure and to increase
among people about the dangers of drinking
passenger fare.
polluted water so that they themselves refrain
II. An amount of about Rs 300 crore from the same, and at the same time taking
should be provided to Vayudoot to make the steps to provide safe drinking water. So, both
airliner economically viable. the courses follow.
3. Statement:
Some serious blunders were detected in the Answer: A
Accounts section of a factory.
Solution:
Courses of Action:
Airlines, being convenient and faster means of
I. An efficient team of auditors should be transport, people would surely prefer it to the
appointed to check the Accounts. railways if there is a marginal difference
between the fares. Hence, a considerable gap
II. A show cause notice should be issued
between the two fares is a must for the
to all the employees involved in the
railways. So, course I follows. Following course
irregularity.
II would reduce the volume of passengers.
A. Only I follows Hence, II does not follow.
Clearly, the situation demands that the faults in ii. Food, water and fodder should
Accounts be properly worked out and the immediately be sent to all these areas to save
persons involved be interrogated about the the people and cattle.
matter. So, both the courses follow. A. Only I follows
4. Statement: B. Only II follows
Due to substantial reduction in fares by C. Either I or II follows
different airline services, large number of
passengers so far travelling by upper classes in D. Neither I nor II follows
trains have switched over to airline services.
E. Both I and II follow
Courses of Action:
Answer: B
I. The railways should immediately
Solution:
reduce the fare structure of the upper classes
substantially to retain its passengers. In the break-out of a natural calamity, the basic
duty of the government becomes to provide
II. The railways should reduce the
the basic amenities essential to save the lives
capacity of upper classes in all the trains to
of people and cattle. Providing financial
avoid loss.
assistance to all would put undue burden on
A. Only I follows the country's resources. So, only II follows.
II. The crude oil prices in the international The two statements discuss two separate
market have gone up substantially in the last statistical and generalised results.
few months.
5. Statements:
A. Statement I is the cause and statement II is
I. The Government has imported large
its effect
quantities of sugar as per trade agreement with
B. Statement II is the cause and statement I is other countries.
its effect
II. The prices of sugar in the domestic
C. Both the statements I and II are market have fallen sharply in the recent
independent causes months.
D. Both the statements I and II are effects of A. Statement I is the cause and statement II is
independent causes its effect
E. Both the statements I and II are effects of B. Statement II is the cause and statement I is
some common cause its effect
C. Either I or II follows
Answer: E
Solution:
5. Statements:
Answer: A
Answer: D
Solution: Number of persons visited
Chidambaram temple on weekend = 10800 –
6400 = 4400 Required difference = (4400 +
4800 + 5200) – 4400 = 10000
A) 9600
B) 6400
a) If the total number of boys and girls in
C) 7200 institute F is 28.56% and 20% more than that
D) 8800 of institute A, then find the average number of
students i n institute F?
Answer: C
A) 375
Solution: Required difference = (7200 + 11600)
- (6400 + 5200) = 7200 B) 325
A) 15%
B) 50%
C) 20%
D) 35%
Answer: B
A) 22
B) 20
C) 26
A) 15% D) 450
B) 75% Answer: B
A) 280 A) 60%
B) 350 B) 25%
C) 600 C) 80%
D) 55% January=100*5/(5-4)=Rs.500 The selling price
of Bata footwear in February=500-100=Rs.400
Answer: A
The selling price of Bata footwear in
Solution: Number of Rs.500 notes printed on March=400+50=Rs.450 Required
Monday=720/3=240 Number of Rs.500 notes ratio=500:450=10:9
printed on Wednesday=560*3/8=210 Number
b) If the selling price of Puma footwear in
of Rs.500 notes printed on Monday and
January is Rs.100 less than the selling price of
Wednesday=240+210=450 Required
VKC footwear in January and the selling price
percentage = ( 720 – 450 ) / 450 * 100 = 270 /
of Puma footwear in February is 8.33% more
450 * 100 = 60%
than that of January. Then fi nd the selling price
4. Read the following information carefully and of VKC footwear in March?
answer the questions:
A) Rs. 440
The given table shows the selling price of five
B) Rs. 460
different footwear i.e. Bata, Nike, Puma, VKC
and Paragon is how much more/less than the C) Rs. 420
selling price of footwear in the previous
D) Rs. 480
month. Note: Assume that the selling price of
footwear every month is more/less than the Answer: D
selling price of footwear in the previous month
Based on their sign(Positive and Negative). Solution: Let the selling price of Puma
footwear in January=12x Let the selling price of
Puma footwear in February=12x*13/12=13x.
The selling price of Puma footwear in
January=12x*25/(13-12)=Rs.300 The selling
price of VKC footwear in
January=300+100=Rs.400 The selling price of
VKC footwear in March=400+100-20=Rs.480
A) Rs. 360
B) Rs. 450
C) Rs. 540
D) Rs. 280
Answer: C
A) Rs. 450
B) Rs. 210
C) Rs. 350
D) Rs. 520
Answer: A
*****
Attention to Detail
Attention to detail questions test your ability A. 12/15/12
to notice small differences, inconsistencies, or B. 30/02/13
patterns. C. c) 31/09/13
D. d) None of the options
Some tips and tricks:
Answer : D
• Slow Down and Focus: Take your time
to avoid missing key details. Explanation:
• Compare Methodically: Check items
By careful observation, we get this answer.
line by line to catch differences.
• Look for Outliers: Identify elements 4. Your answer for each question below would
that don't fit the pattern. be:
• Break Down Complex Information:
A, If ALL THREE items given in the question are
Simplify data into smaller, manageable
exactly ALIKE.
parts.
B, If only the FIRST and SECOND items are
Practice Questions
exactly ALIKE.
1. Which of the following options matches
C, If only the FIRST and THIRD items are exactly
the string given here? AP$%^&1#3
ALIKE.
A) AP$%^1#2
D, If none of the above.
B) AP%$^1#2
ABCD1232YTR3869
c) AP%$^1#2
ABCD1232YTR3689
d) AP$%^&1#3
ABCD1232YTR3869
ANSWER : D
A. A
Explanation:
B. B
By careful observation, we get this answer.
C. C
2. Which of the following options matches
D. D
the number given here? 5478956714098
Answer : C
A) 4478956714098
Explanation:
B) 5478656714098
By careful observation, we get this answer.
C) 5478955714098
5. 98574160323
D) 5478956714098
98574160313
ANSWER: D
98574090323
Explanation:
A. A
By careful observation, we get this answer.
B. B
3. Which of the following dates is in the
format "dd/mm/yy"? C. C
D. D The difference between 2: 31 and 3: 13
is 42 minutes, difference between 3: 13 and 3:
Answer : D
55 means 42 minutes, difference between 3:
Explanation: 55 and 4: 37 means 42 minutes. So, the
difference between 4: 37 and next missing
By careful observation, we get this answer.
time should also 42 minutes. So, 4 : 37 minutes
6. If “/” stands for “+”, “*” stands for “/”, “+” + 42 minutes which equals to 4 : 79 minutes
stands for “-” and “-” stands for “*”, then
calculate 8*5+2/9-1. Hence we will consider 60 minutes equals to 1
hr so 5: 19 minutes will be the answer.
A. 6.8
A. 5.10
B. 5.32
C. 5.13
D. 5.19
How many people are staying at this campsite?
Answer : D
a) 3
Explanation: 2 : 31; 3 : 13; 3 : 55; 4 : 37; ?
b) 6 D. Boat
c) 5 Answer:D
Explanation: - Look at the Board on Duty you 13. Out of the four alternatives given in these
have 4 names and number of plates and questions, which of the following is True?
Spoons are also 4. A. 8+7-6/15*2 = -26.5
B. 10*5/6+20-5 = -92
10. When did they arrive?
C. 50+5-10*2/15 = 40
A. Today D. 14*2+50-10/4 = 75.5
a) Sleeping 55 – 1.33 ≠ 40
b) Running Option D) 14 * 2 + 50 – 10 / 4
c) Trekking 14 * 2 + 50 – 2.5
d) Catching butterflies
28 + 50 – 2.5
Answer :D
78 – 2.5 = 75.5 (Correct)
One person is cooking, another one is taking
pictures, and one person is searching 14. For a beauty pageant, the contestants shall
something in bag, we have a clue in picture be selected for the final round based on the
look at this net and there is butterfly here who following criteria. The candidate:
is trying to catch butterfly.
A. Should be a graduate from any university
12. How did they get there? with minimum 60% marks
A. Walk B. Should possess a height of minimum 5.7 ft.
B. Bus C. Should be well versed in English and any 2
C. Cycle other languages
D. Have a minimum 3 years of full time local languages and has 4 years’ experience in
experience in the modelling field If the the modelling field. She is 5.7 ft. tall. She
contestant satisfies all the criterion she should be:
should be selected, however if a candidate
A. Selected
satisfies all other conditions except:
B. Rejected
A. Condition (i), but has 45% marks in
graduation and has also completed C. Referred to the Secretary of the
master’s degree with minimum 50%, selection board
the candidate must be referred to the
selection board members. D. Data Insufficient
C. Rejected D. D
Answer :D Explanation:
Answer:C S: Subtraction
Explanation: D: Division
A. A C. 10 S 3 M 3 D 7 V 2 M 10 D 2 S7
B. B D. 3 S 10 M 3 D 7 L 10 M 2 D 2 S 7
C. C Answer: C
D. D Explanation:
A: Addition
Cubes and Dices
Basic Concepts in Cube and Dice Problems
Practice Questions D) 1
1. Four positions of a dice are shown below. Answer: A
What number must be at the bottom face
Solution: As both the figure contains one dot
when the dice is in the position as shown in the
and 5 dots, fixing the 5 dots face and moving
figure(iii)
one dot face position to fig (i). we find 4 dots is
opposite to 2 dots.
A) 1
B) 2
C) 4
D) 6
Answer: C
A) Only (i)
B) Only (ii)
A) 3
C) (ii) and (iii) only
B) 4
D) (ii), (iii) and (iv) only
C) 5
Answer: C
D) 6
Solution: option (b) and option (c) cannot
satisfies the conditions of dice formation, as Answer: C
option (b) B cannot be adjacent to D and in Solution: from figure i, ii and iv we
option (c) two white faces cannot be adjacent conclude that 6, 4, 3 and 1 lie adjacent to
to each other. 2. Hence 5 be opposite of 2.
5. Two Position of a dice are shown below as (i)
and (i). When six is at the bottom, what
number will be at the top? 7. How many cubes/blocks are there in below
figure?
A) 1
B) 2
C) 4
A) 8
B) 9
C) 10
D) 11
Answer: B
B) 5
C) 6
d) Cannot be determined
Answer: A
B) 3
C) 4
D) 2
Answer: B
Solution: From all the figures given, we can A) Circle
come to the conclusion that it is not numbers
B) Triangle
5 or 4 or 2 that is opposite to 1. So it will be 3.
C) Dot
D) Cross
11. Three different positions X, Y and Z of a dice
are shown in the figures given below. Which of Answer: B
the hidden numbers adjacent to 5 in position X
is/are common to the hidden numbers Solution: From figures X and Y, we conclude
adjacent to 5 in position Z? that dot, circle, square and cross lie adjacent to
the triangle. Therefore, the arrow must lie
opposite the triangle. From figures X and Z, we
conclude that dot, triangle, arrow and cross lie
adjacent to the circle. Therefore, the square
must lie opposite the circle. Thus, the arrow
lies opposite the triangle, the square lies
opposite the circle and consequently, the cross
lies opposite the dot. As analysed above, the
A) 1 and 4
symbol opposite the arrow is the triangle.
B) 2
13. A dice is numbered from 1 to 6 in different
C) 6 faces. If 1 is opposite to 5 and 2 is opposite to
3, then 4 is opposite to :
D) None
A) 4 is adjacent to 3 and 6
Answer: D
B) 2 is adjacent to 4 and 6
Solution: Since 3 lies opposite 5 (as analysed
above), it follows that 1, 4, 6 and 2 lie adjacent C) 4 is adjacent to 5 and 6
to 5. Out of these four numbers, the hidden
D) 6 is adjacent to 3 and 4
numbers adjacent to 5 in position X are 6 and
2 and the hidden numbers adjacent to 5 in Answer: B
position Z are 1 and 4. Clearly, there is no
Solution: If 1 is opposite to 5 and 2 is opposite
number common.
to 3, then 4 definitely lies opposite to 6.
Therefore, 2 cannot lie opposite to any of the
two numbers - 4 or 6. Hence, 2 necessarily lies
12. A cube has six different symbols drawn
adjacent to both 4 and 6.
over its six faces. The symbols are dot, circle,
triangle, square, cross and arrow. Three 14. How many dots are there on the dice face
different positions of the cube are shown in opposite the one with three dots ?
figures X, Y, and Z. Which symbol is opposite
the arrow?
A) 2
B) 4
C) 5
D) 6 A) 2 Only
Answer: C B) 3 Only
A) 2
B) 4
A) 2
C) 6
B) 3
D) 5
C) 4
Answer: B
D) 5
Solution: In this question, you can see that in
Answer: B figure 1 and 2, 2 and 6 are adjacent to the
Solution: From figures (i) and (ii), we conclude number 3. While in figure 2, 3 is nowhere
that 5, 4, 6 and 2 dots appear adjacent to 3 mentioned. In figure 3 also you will find that
dots. Therefore, 1 dot must appear opposite 3 the 1 and 6 appear adjacent to 3. Thus from the
dots. Thus, if the face with 1 dot is at the figure 1, 2, and 3, you will find that 1, 2, 5, and
bottom, then the face with 3 dots will appear 6 are adjacent to 3. From the options, the only
on the top. number opposite to 3 is 4. Thus option (2) is
the correct answer.
Answer: C
A) 6
B) 4
C) 1
Counting of Geometrical Figures
Basic Concepts
• Break Down Complex Shapes: Divide figures into smaller, recognizable shapes.
• Count Simple Shapes First: Start with the smallest shapes and work up.
• Look for Overlapping Shapes: Be mindful of overlapping figures that might be counted multiple times.
• Use Symmetry: Count one part of symmetrical figures and multiply if needed.
• Number Each Shape: Label or mark shapes as you count to avoid errors.
Practice Questions
A. 16
B. 17
C. 26
D. 30
Answer: D
Solution:
There are 4 rows and 4 columns in the above figure. So let n = 4. Here, we using formulas for finding
number of squares in an n × n grid as follows.
Answer: A
Solution:
Name of the triangle present in the figure are POU, UOQ, QOV, VOR, ROW, WOS, SOT, TOP, SOP, POQ,
QOR, ROS, SPR,QPR, SQP, SQR. Hence there are 16 triangles present in the figure.
A. 20
B. 21
C. 22
D. 23
Answer: B
Solution:
A. 16
B. 17
C. 18
D. 19
Answer: A
Solution:
Total number of Triangles = 6 + (3)2 + 1 = 16
A. 9
B. 10
C. 11
D. 12
Answer: B
Solution:
Name of the triangles are AFG, GBH, HCI, IDJ, JEF, ACJ, CEG, EBI, BDF, ADH Hence, there are 10 triangles
in this figure
A. 8
B. 7
C. 20
D. 5
Answer: A
Solution:
A. 20
B. 30
C. 40
D. 50
Answer: B
Solution:
1 + 4 + 9+ 16 = 30 squares
A. 8
B. 10
C. 12
D. 14
Answer: C
Solution:
Names of the triangles are:
APS, PBQ, QCR, RDS, POS, POQ, ROQ, ROS, PQS, RSQ, PRS, PRO. Hence there are twelve triangles in this
figure.
10. How many lines are there in the given diagram?
A. 14
B. 16
C. 18
D. 12
Answer: 14
Solution:
Horizontal lines = AE, LF, KG = 3 Vertical lines = AK, BJ, CI, DH, EG = 5 Slanting lines = LC, KE, IF, LI, AG,
CF = 6
Therefore, Total number of lines =3 + 5 + 6 + = 14
• This section tests only your ability to Combining both the statements, even if the
determine if the given data is sufficient to class VIII forms 25% of the total strength, the
answer a question and not the exact answer to average age will be less than 17 years.
the question
This is shown below:
Practice Questions
Let the strength of the school be 10k and that
Directions: The following questions will have of the class VIII students be 2.5k. In that case
two statements. Based on the statements, the
select one of the options that follow the
strength of the remaining classes is 7.5k.
questions:
Hence, the average age of the school
1. Is the average age of the students of a school
→ 2.5k × 18 + 7.5k × 16/10k = 16.5 years
less than 17 years?
Hence, the average age will be less than 17
Statement I : The strength of the class VIII is less
years. If the strength of the VIII class is less than
than 25% of the strength of the school.
25%, the average age would be less than 16.5
Statement II : The average age of the students
years.
of class VIII of the school is 18 years and that of
the remaining classes is 16 years. Hence, option D is correct.
A. If the data in statement I alone is sufficient 2. Among 20 retired persons, is the number of
to answer the question, while the data in persons, who are having at least 20 years of
statement II alone is not sufficient to answer service, less than 8?
the question
Statement I : Exactly 17 persons joined the
B. If the data in statement II alone is sufficient service before 2000 and exactly 14 persons
to answer the question, while the data in retired after 2020.
statement I alone is not sufficient to answer the
Statement II : Exactly 14 persons joined the
question
service after 2000 and exactly 6 persons retired
C. If the data either in statement I alone or in before 2020.
statement II alone is sufficient to answer the
A. If the data in statement I alone is sufficient
question
to answer the question, while the data in
D. If the data in both statements I and II statement II alone is not sufficient to answer
together are necessary to answer the question the question
B. If the data in statement II alone is sufficient Hence, option A is correct.
to answer the question, while the data in
3. The lengths of trains A and B are in the ratio
statement I alone is not sufficient to answer the
2:3. Which of them takes less time to cross the
question
same platform?
C. If the data either in statement I alone or in
Statement I : The time taken by train A to cross
statement II alone is sufficient to answer the
train B when they are moving in opposite
question
directions is half the time taken by it to cross
D. If the data in both statements I and II train B, when moving in the same direction.
together are necessary to answer the question
Statement II : The ratio of the length of train B
E. If the data given in both statements I and II to that of the platform is 4 : 3.
together are not sufficient to answer the
A. If the data in statement I alone is sufficient
question.
to answer the question, while the data in
Answer: A statement II alone is not sufficient to answer
the question
Explanation:
B. If the data in statement II alone is sufficient
From I alone,
to answer the question, while the data in
Joined statement I alone is not sufficient to answer the
question
2000 or before = 17
C. If the data either in statement I alone or in
2020 or before = 6 statement II alone is sufficient to answer the
Retired : question
4. Is the speed of the boat in still water at least Also, we have upstream speed of the boat 16/4
twice that of the speed of the stream? = 4 kmph
Statement I : The time taken by the boat to Now, x and y can be compared.
reach a point P, from Q is exactly twice the time
Hence, option C is correct.
taken by the boat, to reach Q from the point P.
5. Rani, Supriya and Amita work on a project. If
Statement II : The time taken by the boat to
each of them is of a different efficiency. Who is
cover 8 km downstream is 40 minutes and it
the fastest worker among them?
takes four hours to cover 16 km upstream.
Statement I : Rani and Supriya take 5 days to
A. If the data in statement I alone is sufficient
complete the project while Supriya and Amita
to answer the question, while the data in
take 10 days to complete the same project.
statement II alone is not sufficient to answer
the question Statement II: Rani and Amita take 30/7 days to
complete the project.
B. If the data in statement II alone is sufficient
to answer the question, while the data in A. If the data in statement I alone is sufficient
statement I alone is not sufficient to answer the to answer the question, while the data in
question statement II alone is not sufficient to answer
the question
C. If the data either in statement I alone or in
statement II alone is sufficient to answer the B. If the data in statement II alone is sufficient
question to answer the question, while the data in
statement I alone is not sufficient to answer the
D. If the data in both statements I and II
question
together are necessary to answer the question
C. If the data either in statement I alone or in
E. If the data given in both statements I and II
statement II alone is sufficient to answer the
together are not sufficient to answer the
question
question.
D. If the data in both statements I and II
Answer: C
together are necessary to answer the question
Explanation:
E. If the data given in both statements I and II
Let the speed of the boat in still water be x together are not sufficient to answer the
kmph and that of the stream be y kmph. From question.
statement I,
Answer: A
we have the boat moving opposite the stream.
Explanation:
Also let the distance between P and Q be d.
Then, From statement I, Rani and Supriya take 5 days
to complete the project implies one of them
2 d/(x+y) = d / (x – y)
(Rani or Supriya) takes less than 10 days and
→ x = 3y the other takes more than 10 days. Supriya and
Amita take 10 days to complete the project
implies one of them ( Supriya or Amita ) takes Ratio of ages of Sonia and Julia 6 years ago was
less than 20 days and the other takes more 9 : 7, i.e.
than 20 days. Supriya cannot take less than 10
⇒ s – 6/ j – 6 = 9/ 7
days because if Supriya takes less than 10 days
than Supriya and Amita should take less than From statement II :
10 days to complete the project.(but given
Supriya and Amita take 10 days to complete the Ratio of ages of Julia and Amelia after 10 years
project). Rani takes less than 10 days and she is will be 6 : 7, i.e.
the fastest. From statement II, we do not know ⇒ j + 10/ a + 10 = 6/7
anything about Supriya, so who is fastest
cannot be determined. We can’t solve the question with these two
statements.
Hence, option A is correct.
Hence, option E is correct.
6. Sum of ages of Julia, Rani, Amelia and Sonia
is 91. What is the present age of Amelia? 7. What will be the height of three friends
Rahul, Robert and Priyanka altogether, if
Statement I : Ratio of ages of Sonia and Julia 6 Priyanka’s height is 39 cm?
years ago was 9 : 7.
Statement I : The sum of the heights of Rahul
Statement II : Ratio of ages of Julia and Amelia and Priyanka is equal to 160 cm.
after 10 years will be 6 : 7.
Statement II : The height of Robert is equal to 3
A. If the data in statement I alone is sufficient times the height of Priyanka.
to answer the question, while the data in
statement II alone is not sufficient to answer A. If the data in statement I alone is sufficient
the question to answer the question, while the data in
statement II alone is not sufficient to answer
B. If the data in statement II alone is sufficient the question
to answer the question, while the data in
statement I alone is not sufficient to answer the B. If the data in statement II alone is sufficient
question to answer the question, while the data in
statement I alone is not sufficient to answer the
C. If the data either in statement I alone or in question
statement II alone is sufficient to answer the
question C. If the data either in statement I alone or in
statement II alone is sufficient to answer the
D. If the data in both statements I and II question
together are necessary to answer the question
D. If the data in both statements I and II
E. If the data given in both statements I and II together are necessary to answer the question
together are not sufficient to answer the
question. E. If the data given in both statements I and II
together are not sufficient to answer the
Answer: E question.
Explanation: Answer: D
Sum of ages of Julia, Rani, Amelia, and Sonia is Explanation:
91, i.e.
From statement I :
⇒ j + r + a + s = 91
Rahul + Priyanka = 160 cm .......... (i)
From statement I:
From statement II : Selling price = Marked Price (1 – Discount%) =
Marked price (1 – 21%)
Robert = 39 × 3 = 117 cm ..............(ii)
Now if we combine Statement I & statement II
Adding equations (i) and (ii)
Selling price = (1.4X) × (79/100)
Rahul + Priyanka + Robert = 160 + 117 = 277 cm
Profit (Gain) = 1.106X
∴ Both statements are required to answer the
question. Gain percentage = 10.6%
Explanation: Answer: D
Then, Marked price = 140% of X = 1.4X The product of the digit is 15 and the difference
between the digits is 2.
From statement II :
⇒ m × n = 15 …(1) Explanation:
⇒ m – n = 2 or n - m = 2 From statement I:
Putting these two values in equation 1 we get; ∴ In 1 hour it can fill 1/12th part of the tank.
∴ Statement I is not sufficient to reach at the Tap T2 can empty ‘50%’ the tank in 10 hours
solution.
It can empty the full tank in 20 hours
From statement II:
∴ In 1 hours it can empty 1/20th part of the
The digit at the unit’s place is greater than the tank
other.
Combining the two statements:
Statement II alone is also not sufficient.
The part of the tank filled in 1 hour
But using both, we can get the answer.
=1/12 – 1/20 = (5-2)/20 = 1/30th
Hence, option D is correct.
∴ The tank will be filled in 30 hours.
10. If two taps, T1 and T2 are opened
∴ 50% tank will be filled in 15 hour.
simultaneously, how long would it take for the
empty tank to be filled to 50% of its capacity? ∴ Both statements are required to answer the
question.
Statement I : T1 can fill the empty tank in 12
hours. Hence, option D is correct.
Statement II : T2 can empty 50% the tank in 10 *****
hours.
Answer: D
Coding & decoding, Series, Analogy, Odd man out, and Visual reasoning
'236' means 'one red carpet'. Which digit in
that code means 'dust'?
Concepts to remember
A. 2
• While answering questions on coding and
B. 3
decoding, do not decide the pattern by just
C. 5
checking 2 or 3 terms. Check all the terms for
D. 6
the pattern.
Answer: C
• In visual reasoning, to find the pattern, follow
the position and direction of one element at a Explanation:
time. Trying to follow all the elements at the In the first and second statements, the
sametime could lead to confusion. common code digit is '2' and the common word
is 'carpet'.
• Never assume a pattern to answer the So, '2' means 'carpet'.
questions. In the second and third statements, the
• In questions on coding - decoding, check common code digit is '6' and the common word
carefully if the final answer is for the coded or is 'one'.
decoded work. So, '6' means 'one'.
• In analogy, remember the relation between Therefore, in the second statement, '5' means
first two terms should be the same for the next 'dust'.
two terms. 3. If train is called bus, bus is called tractor,
Practice Questions tractor is called car, car is called scooter,
scooter is called bicycle, bicycle is called
1. In a certain language, if ENTRY is coded as moped, which is used to plough a field?
12345, and STEADY is coded as 931785, then
which is the correct code for the word below? A. Train
B. Bus
NEATNESS C. Tractor
D. Car
A. 25196577
B. 21732199 Answer: D
C. 21362199
D. 21823698 Explanation:
A ’tractor’ is used to plough a field.
Answer: B But a ’tractor’ is called ’car’.
So, ’car’ will be used to plough the field.
Explanation:
N is coded as 2, 4. If in a certain code language, 'oka peru'
E as 1, means 'fine cloth' ; 'meta lisa' means 'clear
A as 7, water' and 'dona lisa peru' means 'fine clear
T as 3 and weather' , which word in that language means
S as 9. 'weather' ?
So, NEATNESS is coded as 21732199.
A. peru
2. In a certain code, '247' means ’spread red B. oka
carpet’; '256' means 'dust one carpet' and C. meta
D. dona
Answer: D A. 321326
B. 312326
Explanation:
C. 326321
In the first and third statements, the common
D. 362312
code word is ’peru’ and the common word is
’fine’. Answer: A
So, ’peru’ means ’fine’.
Explanation:
Similarly, in the second and third statements, By observing the coding of the words, each
the common code word is 'lisa' and the alphabet corresponds to a certain place value
common word is 'clear'. or a number.
Explanation: 1 1 6 8 1 2 5?
In the second and third statements, the A. 6
common code is ’9a’ and the common word is B. 8
’not’. C. 4
So, ’9a’ means ’not’. D. 5
In the first and second statements, the
common codes are ’7c’ and ’3a’ and the Answer: A
common words are ’is’ and ’Eternal’. Explanation:
So, in the second statement, ’8b’ means
’enmity’. By looking at the pattern it is of the form
14,24,34 and the next term must be 44 ie,256 so
16. Find out the wrong term in the series: the next term is 6.
2, 3, 4, 4, 6, 8, 9, 12, 16 19. Find the missing term in the given series.
A. 9 0, 3, 7, 10, 1?
B. 8
C. 4 A. -64
D. 16 B. -84
C. -48
Answer: A D. -24
Explanation: Answer: B
The given sequence is a combination of three Explanation:
series
It follows the pattern (n3-n!)/n and n values 5 10?
must start from 1,2,3...
A. 15
So the last value is (63-6!)/6 = -84. B. 20
C. 25
20. 88, 99, 92, 97, 94?
D. 50
A. 98
Answer: B
B. 92
C. 96 Explanation: Each number is multiplied by 2.
D. 102
23. Find the next term of the given series. 17,
Answer: C 18, 19, 34, 35, 19, 20, ___
⇒ 92 + 5 = 97 Answer: C
⇒ 97 - 3 = 94 Explanation:
Hence, the missing term is 96 The numbers inside brackets are consecutive
numbers. Following this, the missing number
21. 7, 10, 8, 11, 9, 12? What number should will be 21.
come next?
24. Find the wrong number in the series: 4, 9,
A. 13 25, 49, 124, 169
B. 10
C. 14 A. 124
D. 16 B. 169
C. 49
Answer: B D. 25
Explanation: Answer: A
This is a simple alternating addition and Explanation: All the numbers are squares of the
subtraction series. prime numbers, except 124. Hence, the odd
In the first pattern, 3 is added; term is 124.
In the second pattern, 2 is subtracted.
25. Find the wrong number in the series:
Rule =(1st number) (1st number + 3 = 2nd 1,3,7,13,21,33,43
number) (2nd number - 2 = 3rd number) (3rd
number + 3 = 4th number) A. 43
Now (7) (7 + 3 = 10) (10 - 2 = 8) (8 + 3 = 11) (11 B. 33
- 2 = 9) (9 + 3 = 12) (12 - 2 = 10)... C. 13
So the answer is b = 10. D. 21
3 6 12
248
Explanation: The pattern followed is +2, +4, +6, B. 120
+8, +10, +12. So, it should have been 31 in place C. 130
of 33. D. 114
Answer: D
A. 190 A. 225
B. 22 B. 144
C. 22 C. 121
D. 380 D. 129
Answer: D Answer: A
Explanation: Explanation:
784 : 812 :: 1024 : __ 38. Find the missing term in the following
series:
A. 1040
B. 1056 U : X :: ? : M
A. K Answer: B
B. M
Explanation: Given F : L :: I : ? In alphabet series
C. O
F is the 6th letter and L is the 12th letter. i.e (6
D. J
* 2 = 12)
Answer: D
Similarly I is the 9th letter, hence (9 * 2 = 18)
Explanation: 18th letter is R
Explanation: 3⁴ = 81
(2)3=2×2×2 = 8 5⁴ = 625
28 = (3)3+1=3×3×3 + 1 = 28 .
6⁴ = 1296
? = 4×4×4 + 1 = 65
74= 2401
46. Fill the missing term: 2918 : 1808 :: 6744 : ?
49. Directions for questions: Identify the odd
A. 4212 term.
B. 5616
12, 24, 48, 56, 60, 72
C. 6318
D. 4216 A. 24
B. 48
Answer: D
C. 60
Explanation: Product of first two numbers of 556
the first term forms first two numbers of
Answer: D
second term. 2*9=18 : 1*8=08 ==>1808
Explanation: All numbers are a multiple of 6
Similarly the product of next two numbers of
while 56 is not.
first term forms the next two numbers of next
term.i.e.,6*7=42 4*4=16 ==>4216 20. Directions for questions: Identify the odd
term.
47. Directions for questions: Identify the odd
term. 132, 264, 396, 572, 671, 789
2, 4, 8, 14, 19, 20, 24 A. 671
B. 789
A. 14
C. 572
B. 20
D. 264
C. 19
D. 24 Answer: B
Answer: C Explanation: 789 is the odd number, because in
each number except 789, the middle digit is the
Explanation: 19 is an odd number while others
sum of the other two.
are even.
Hence, 789 is the answer.
48. Directions for questions: Identify the odd
term. 51. Directions for questions: Identify the odd
term.
1, 16, 81, 256, 625, 1236, 2401
8, 20, 32, 44, 56, 87
A. 1236
B. 625 A. 32
C. 2401 B. 44
D. 256 C. 58
D. 87
Answer: A
Answer: D
From the given series, we observe that
Explanation: Each of the numbers except 87, is
1⁴ = 1
a multiple of 4. So 87 is the answer.
52. Directions for questions: Identify the odd D. 369
term.
Answer: A
1, 16, 36, 64, 100, 124
Explanation: 282 is the odd one. Because this
A. 124 pattern follows adding of 52 and subtraction of
B. 100 63. Hence 282 is the answer.
C. 64
56. Which figure is the odd one out?
D. 36
Answer: A
A. 11 Answer : D
B. 19
Explanation :
C. 32
Consider a circle as having one "side" as it does
D. 41
not have end-points.
Answer: C
Question Figure A: Pentagon + Triangle. Total
Explanation: The pattern follows (1*1)+0, sides = 5+3 = 8
(2*2)+1, (3*3)+2, (4*4)+3, (5*5)+4, (6*6)+5
Question Figure B: 2 squares. Total sides = 4+4
32 is out of pattern. =8
54. Directions for questions: Identify the odd Question Figure C: 2 Triangles + 2 circles. Total
term. sides = 3+3+1+1 = 8
211, 312, 431, 547, 615, 743 Question Figure D: 1 square + 1 circle. Total
sides = 4+1 = 5
A. 431
B. 547 Hence, figure D is the incorrect member.
C. 615
57. Find out from amongst the four
D. 743
alternatives as to how the pattern would
Answer: B appear when the transparent sheet is
folded at the dotted line.
Explanation: In each number except 547, the
difference of first and third digits is the middle
one. Hence 547 is the odd one.
A. 1
B. 2
C. 3
D. 4
Answer : D
Explanation :
Answer : D The fourth alternative is the answer,
Explanation : because it is the mirror image of the
Figure 2 is the mirror image of Figure 1 given statement.
Figure 3 is the water image of Figure 2
61. Which figure is the odd one out?
Figure 4 is the mirror image of Figure 3
So there is an alternating pattern of mirror
images and water images of the previous
figure, which is followed.
Hence, Figure 5 has to be the water image of
Figure 4. A. A
Hence, figure 4 is the answer. B. B
C. C
59. Which figure is the odd one out? D. D
Answer : B
Explanation :
In this pattern, the round mark always
be in the layer of square. But that
A. A pattern does not follow in the figure b.
B. B Hence it is the answer.
C. C
D. D 62. Choose the correct water - image of the
main figure from the given four figures.
Answer : D
Explanation :
Comparing figure a and figure b the closed part
rotates in clockwise direction. Then from figure
b and figure c it rotates in clockwise direction.
A. 1
B. 2
C. 3
D. 4
Answer : C
A. 1
B. 2
C. 3
D. 4
Answer : D
A. 1
B. 2
C. 3
D. 4
Answer : C
Answer: B
Explanation: 5 symbols are arranged in 5
possible positions, so 6th image should start as
like the 1st arrangement.
*****
Critical Reasoning(Premise, Assumption, Conclusion, Mimic the pattern)
The passage has no reference to the speeds at Choice (a) supports the salesman’s statement.
which vehicles are driven on roads having It states that those who do not watch TV tend
heavy traffic. Also the words "mostly on the to spend less. This supports the salesman’s
roads having scanty traffic" does not rule not statement.
two wheelers being driven at 50 km/hr and Choice (b) is referring to the TV signals but not
above speeds on roads having heavy traffic. about advertisements. Hence, (b) is out of
Hence, (b) cannot be concluded. context.
Driving at speeds above 50 km/hr does not Choice (c) states that the cable TV channels
necessarily mean that the driver is reckless. also allow advertisements as much as free TV.
Hence, (c) cannot be concluded. Since the salesman has pointed out that
Since it is said that generally accidents advertisement make viewing of free TV
involving two wheelers take place when they indirectly costly, the same is applicable to
are driven at speeds above 50 km/hr and they cable TV as well.
are driven at the speed on roads having scanty Hence, (c) goes against the statement of the
traffic, it can be concluded that most of such salesman.
According to choice (d) people get access to there are many supermarkets in the city which
some free TV channels also. But it is not clear sell food at cheaper prices, and many of these
whether those few free channels have supermarkets are open 24-hours.Which of the
advertisements. following, if true, would help the most in
Hence, (c) does against the statement of the explaining the paradoxical observation?
salesman.
A. The corner shops are open even during
6.Arun has twenty years of typing experience nights.
behind him; therefore, if you are looking for an
B. The supermarkets are mostly located
efficient typist to enter your data into the new
on the outskirts of the city and require
system, you need look no further.The speaker
residents to use cars or public
assumes that
transport to reach them.
A. Twenty years of practice ensures
C. The main business of the corner shops
typing efficiency
is selling food items.
B. The type of typing required for the
D. The corner shops are willing to make
new system is identical to what Arun
home deliveries
has been doing
Answer: B
C. Arun’s job profile is the best that the Explanation:
new employer is going to get
First look for the statements that help explain
D. Arun is an outstandingly fast and
why there continues to be a large number of
accurate typist
corner shops. Choice "The corner shops are
open even during nights" suggests that they
Answer: A
stock different things; "The main business of
Explanation:
the corner shops is selling food items" suggests
All that the speaker implies is that Arun is their main business does not compete with
efficient because he has twenty years of supermarkets; "The corner shops are willing to
practice, and so answer ''Twenty years of make home deliveries" suggests that they find
practice ensures typing efficiency'' is correct. favor with the community who would be likely
Eliminate ''The type of typing required for the to patronize them. "The supermarkets are
new system is identical to what Arun has been mostly located on the outskirts of the city and
doing'' because the word ‘identical’ is not require residents to use cars or public
implied. Eliminate ''Arun’s job profile is the transport to reach them" suggests that they
best that the new employer is going to get'' are more conveniently located. So this is the
because we are talking about typing not the correct answer .
whole ‘job profile’. The argument claims that
Arun is efficient but without defining efficiency
we cannot assume that it involves both speed 8.The average normal infant born in the India
and accuracy, and certainly ‘outstandingly’ is weighs between 6 and 8 kilograms at the age
not warranted. Nothing is implied about fitting of three months. Therefore, if a three-month-
into the office. old child weighs only 5 kilograms, its weight
gain has been below the Indian average.Which
7.It is strange that in Mancity there are so of the following indicates a flaw in the
many corner shops selling food items. After all reasoning above?
A. Weight is only one measure of normal D. If it is windy and not sunny then Arjun
infant development. goes hunting
B. Some three-month-old children weigh Answer: B
as much as twelve kilograms. Explanation:
C. It is possible for a normal child to Very simple question, just by reading we can
weigh five kilograms at birth. find the answer . Statement "If it is sunny and
D. Average weight gain is not the same as Arjun does not go fishing then it is windy"
average weight indirect way of saying what is given in the
question. So this is the correct answer.
Answer: D
Explanation: 10.Increase in the level of high-density
lipoprotein (HDL) in the human bloodstream
The evidence on which the conclusion is based lower bloodstream-cholesterol levels by
concerns only average weight, but the increasing the body’s capacity to rid itself of
conclusion concerns average weight gain. The excess cholesterol. Levels of HDL in the
relevant reasoning error is described in bloodstream of some individuals are
"Average weight gain is not the same as significantly increased by a program of regular
average weight", which is the best exercise and weight reduction.Which of the
answer. "Weight is only one measure of following can be correctly inferred from the
normal infant development" identifies a statements above?
reasoning error in the passage, since the
passage makes no claim that weight is the only
A. Individuals who are underweight do
relevant measure of infant development in
not run any risk of developing high
general ." Some three-month-old children
levels of cholesterol in the
weigh as much as twelve kilograms" and "It is
bloodstream.
possible for a normal child to weigh five
kilograms at birth" are consistent with the B. Individuals who do not exercise
claims in the passage, and neither identifies a regularly have a high risk of developing
flaw in the argument. high levels of cholesterol in the
bloodstream late in life.
9.Arjun goes either hunting or fishing every C. Exercise and weight reduction are the
day. If it is snowing and windy then Arjun goes most effective methods of lowering
hunting. If it is sunny and not windy then Arjun bloodstream cholesterol levels in
goes fishing. Sometimes it can be snowing and humans.
sunny.Which of the following statements must D. A program of regular exercise and
be true? weight reduction lowers
E. cholesterol levels in the bloodstream
A. If it is snowing and sunny then Arjun of some individuals.
goes hunting Answer: D
B. If it is sunny and Arjun does not go Explanation:
fishing then it is windy.
If increased HDL levels cause reduced
C. If it is not sunny and it is snowing then cholesterol levels and if a certain program
Arjun goes hunting increases HDL levels in some individuals, it
follows that some individuals who undertake
that program achieve reduced cholesterol educational software programs are
levels. " A program of regular exercise and more marketable that are books and
weight reduction lowers cholesterol levels in articles.
the bloodstream of some individuals." is thus Answer: B
correctly inferable and the best answer. Explanation:
"Individuals who are underweight do not run
The passage concludes that, where royalty
any risk of developing high levels of cholesterol
retention of faculty members’ works is
in the bloodstream" cannot be correctly
concerned, software should be treated as
inferred because the statements do not
books and articles are, not as inventions are.
establish any connection between being
The conclusion requires an additional
underweight and levels of cholesterol.
premise establishing that software is, in
"Individuals who do not exercise regularly have
relevant respects, more comparable to books
a high risk of developing high levels of
and articles than to inventions. "In terms of the
cholesterol in the bloodstream late in life" is
criteria used to award royalties, educational
incorrect, since there is no indication that
software programs are more nearly
exercise alone is either necessary or sufficient
comparable to books and articles than to
to decrease cholesterol levels. "Exercise and
inventions" provides this kind of premise and
weight reduction are the most effective
is therefore the best answer. "Royalties from
methods of lowering bloodstream cholesterol
inventions are higher than royalties from
levels in humans" is inappropriate because
educational software programs", "Inventions
other methods of cholesterol reduction are not
bring more prestige to universities that do
addressed.
books and articles" and "In the experience of
11.Even though most research institutes retain most universities, educational software
the royalties from faculty members’ programs are more marketable that are books
inventions, the faculty members retain the and articles" each describe some
royalties from books and articles they write. difference between software and inventions,
Therefore, faculty members should retain the or between software and books and articles.
royalties from the educational computer However, none establishes the required
software they develop.The conclusion above relationship among inventions, software,
would be more reasonably drawn if which of books and articles .
the following were inserted into the argument
12.Stringent hiring standards have not been
as an additional premise?
the primary cause of the present staffing
A. Royalties from inventions are higher shortage in government schools. The shortage
than royalties from educational of teachers is primarily caused by the fact that
software programs. in recent years teachers have not experienced
B. In terms of the criteria used to award any improvements in working conditions and
royalties, educational software their salaries have not kept pace with salaries
C. programs are more nearly comparable in other professions.Which of the following, if
to books and articles than to true, would most support the claims above?
inventions. A. Many teachers already in the
profession would not have been hired
D. Inventions bring more prestige to
universities that do books and articles. under the new hiring standards.
B. Some teachers have cited higher
E. In the experience of most universities,
standards for hiring as a reason for the than to produce television in Country B.The
current staffing shortage. statements above, if true, best support which
C. Many teachers have cited low pay and of the following assertions?
lack of professional freedom as A. labor costs in Country A are ten
reasons for their leaving the percent below those in Country B.
profession. B. Importing televisions from Country A
D. Many prospective teachers have cited to Country B will eliminate ten percent
the new hiring standards as a of the manufacturing jobs in Country
E. reason for not entering the profession B.
Answer: C C. "The fee for transporting a television
Explanation: from Country A to Country B is more
than ten percent of the cost of
The passage rejects one explanation of the
manufacturing the television in
shortage of teachers-that it results from
Country A"
toughened hiring standards-and advances an
alternative-that it results from deficiencies in D. The tariff on a television imported
pay and wording conditions." Many teachers from Country A to Country B is less
have cited low pay and lack of professional than twenty percent of the cost of
freedom as reasons for their leaving the manufacturing the television in
profession " provides corroborative evidence Country B.
for the latter explanation by suggesting that, Answer: D
for many former teachers, poor pay and Explanation:
working conditions were reasons for their If the tariff on importing radios from Country A
quitting the profession. Therefore, "Many to Country B were as high as ten percent or
teachers have cited low pay and lack of more of the cost of producing radios in B, then,
professional freedom as reasons for their contrary to what the passage says, the cost of
leaving the profession" is the best answer. " importing radios from A to B would be equal to
Many teachers already in the profession would or more than the cost of producing radios in B.
not have been hired under the new hiring thus, the tariff cannot be that high, and "The
standards" ," Some teachers have cited higher tariff on a television imported from Country A
standards for hiring as a reason for the current to Country B is less than twenty percent of the
staffing shortage" and " Many prospective cost of manufacturing the television in Country
teachers have cited the new hiring standards B'' is the best answer. " labor costs in Country
as a reason for not entering the A are ten percent below those in Country B "
profession" provide evidence that tends to give possible partial explanations for the cost
implicate new hiring standards in the staffing difference, but is not supported by the passage
shortage, and thus support the explanation because the cost advantage in A might be
that the passage rejects. attributable to other factors. "Importing
13.The cost of producing television in Country televisions from Country A to Country B will
A is twenty percent less than the cost of eliminate ten percent of the manufacturing
producing television in Country B. Even after jobs in Country B" and "The fee for
transportation fees and tariff charges are transporting a television from Country A to
added, it is still cheaper for a company to Country B is more than ten percent of the cost
import television from Country A to Country B of manufacturing the television in Country
A" are both consistent with the information in None of the choice (a), (c) and (e) has anything
the passage, but the passage provides no about the cost. Hence, none of them weakens
evidence to support them. the conclusion.
Choice (b) states that the new products can be
14.We, at Comfort Stationers, have always manufactured and they also last longer. This
been striving to provide stationery items that information strengthens the conclusion.
would make your work more enjoyable and According to choice (d) the procurement and
less strenuous. Our latest innovations are a storage costs are very high. This casts a doubt
smooth-flow pen and gradual-friction paper. A on the conclusion that the cost of clerical job
combination of these two reduces strain on can be decreased by using these new products.
your fingers and allows faster writing that Hence, (d) weakens the conclusion.
causes lesser fatigue. Therefore, replacement
of your pen and paper with our innovative 15.Kishore: I sold my land on an internet site
products reduces cost of clerical jobs. last month and was happy with the price. I got
Which of the following, if true, a speedy sale and the cost of advertising was
would weaken the conclusion negligible. I would advise you to avoid real
drawn in the above argument? estate agents.
A. Those who are already using the above Bala: It is in the interest of the real estate agent
new products report greater difficulty to get me the best price for my land because
in transition from new products to he gets a commission based on the selling
regular ones than from the regular price. Therefore, when selling my land I will
ones to the new ones. certainly use an agent rather than trying to sell
the land on the internet.Bala could strengthen
B. The cost of manufacturing these new
his position by pointing out all of the following
products is not more than the cost of
except .
manufacturing the regular ones and
the new products last longer than the
regular ones. A. Lands of comparable value often
obtain a lower price when sold on the
C. The number of offices using the new
internet
products is increasing month by
month. B. Very few houses are sold on the
internet at the moment and so a
D. These products need to be purchased
in huge lots and need to be stored in C. valid comparison is difficult
special conditions. The cost of D. The agent’s service includes many add-
procurement and strong is quite high on benefits in terms of legal fees,
Answer: D surveyor’s reports and advice that are
Explanation: not available on internet sites
The passage talks about the advantages in E. Some buyers pay the agent to find
writing by using the new kind of pen and paper them a cheap house
and arrives at a conclusion about the cost of Answer: D
clerical job. We have to show that the new Explanation:
products do not lead to decrease in cost of Since this is an ‘except’ question we must find
clerical job. four ways for Bala to strengthen his position.
The one answer that doesn’t strengthen his
conclusion will be correct. Bala wants to use an country consumes could reduce the economic
agent and choices ''The agent’s service impact of these increases. "Decreasing oil
includes many add-on benefits in terms of legal consumption through conservation" gives a
fees, surveyor’s reports and advice that are not way to reduce oil consumption and is thus the
available on internet sites'' point out benefits best answer. "Maintaining the quantity of oil
of the agent, while choices "Lands of imported at constant yearly levels" and
comparable value often obtain a lower price "Decreasing domestic production of oil."
when sold on the internet" and ''Very few describe policies that could actually increase
houses are sold on the internet at the moment the long-term impact of increases in
an so a valid comparison is difficult" point to international oil prices, so neither of these
reasons why the internet is not necessarily choices is appropriate. No relationship
better. A careful look at "Some buyers pay the is established between the economic impact
agent to find them a cheap house" suggests and either the number of oil tankers or
that the agent might not always get the seller diplomatic relations in "Increasing the number
the best price, and so that is the best answer. of oil tankers in its fleet" and "Suspending
diplomatic relations with major oil-producing
16.If there is an oil-supply disruption resulting nations.", so neither of these choices
in higher international oil prices, domestic oil is inappropriate.
prices in open-market countries such as the
United States will rise as well, whether such 17.Bella: The alarming fact is that 95 percent of
countries import all or none of their oil.If the the people in this country now report that they
statement in the passage concerning oil-supply know someone who is unemployed.Shashi: But
disruptions is true, which of the following a normal, moderate level of unemployment is
policies in an open-market nation is most likely 5 percent, with 1 out of 20 workers
to reduce the long-term economic impact on unemployed. So at any given time if a person
that nation of sharp and unexpected increases knows approximately 50 workers, 1 or more
in international oil prices? will very likely be unemployed.Shashi’s
argument is structured to lead to which of the
A. Maintaining the quantity of oil following as a conclusion?
imported at constant yearly levels A. The fact that 95% of the people know
B. Increasing the number of oil tankers in someone who is unemployed is not an
its fleet indication that unemployment is
C. Suspending diplomatic relations with abnormally high.
major oil-producing nations. B. The current level of unemployment is
D. Decreasing oil consumption through not moderate
conservation C. If at least 5% of workers are
Answer: D unemployed, the result of questioning
Explanation: a representative group of people
cannot be the percentage Roland cites.
If the statement about oil-supply disruption is
true, domestic oil prices in an open-market D. It is unlikely that the people whose
country will rise when an oil-supply disruption statements Roland cites are
causes increased international oil prices. A E. giving accurate reports
reduction in the amount of oil an open-market Answer: A
Explanation: answer is "The dogs will form an
uncontrollable pack" this would cast the most
Referring from the options we can directly
serious doubt on the prospects for success of
eliminate option 3 and 4 since there is no
the corporation’s plan.
reference about roland cities in the argument.
Option 2 cannot be the answer since we
19.Citizens of Scotland are worried by the
cannot conclude the certainity of the
increased frequency of serious crimes
population as moderate. Hence the correct
committed by local teenagers. In response, the
option is" The fact that 95% of the people
government has instituted a series of
know someone who is unemployed is not an
measures designed to keep teenagers at home
indication that unemployment is abnormally
in the late evening. Even if the measures
high."
succeed in keeping teenagers at home,
however, they are unlikely to affect the
18.Birds that gather at the pond of a large
problem that concerns citizens, since most
corporation create a hazard for executives who
crimes committed by local teenagers take
use the corporate helicopter, whose landing
place in the afternoon.
site is 30 feet away from the pond. To solve the
Which of the following, if true, most
problem, the corporation plans to import a
substantially weakens the argument?
large number of herding dogs to keep the birds
away from the helicopter.Which of the A. Similar measures adopted in other
following, if a realistic possibility, would cast place have failed to reduce the no of
the most serious doubt on the prospects for teenagers in the late evening.
success of the corporation’s plan? B. Crimes committed by teenagers in
afternoon are mostly small thefts and
inconsequential vandalism.
A. The dogs will form an uncontrollable
pack. C. Teenagers are much less likely to
commit serious crimes when they are
B. The dogs will require training to learn
at home than when they are not at
to herd the birds.
home.
C. The dogs will frighten away foxes that
D. Any decrease in the need for police
prey on old and sick birds.
patrols in late evening would not mean
D. Some of the birds will move to the that there could be more intensive
pond of another corporation in order patrolling in the afternoon.
to avoid being herded by the dogs. Answer: B
Explanation:
Answer: A
Explanation: Problem is that city people are concerned with
the increase of frequency in serious crimes.
"The dogs will require training to learn to herd The word serious is the key here. Conclusion is
the birds" even if they were to be trained even that the measure to keep children after late
then does not make sense so eliminate evening will not help because most crime
this." The dogs will frighten away foxes that happens in the afternoon. We have to prove
prey on old and sick birds " and " Some of the that measures will be effective. What if the
birds will move to the pond of another crimes that happen in the afternoon are petty
corporation in order to avoid being herded by and inconsequential? Not serious any more??
the dogs " both are out of context. correct
measures are effective. Hence " Crimes Sharks that prey on dolphins are less common
committed by teenagers in afternoon are in the open seas off the beach than in many
mostly small thefts and inconsequential other areas of the open seas where dolphins
vandalism" is the correct answer. Other congregate" is exactly the opposite of what we
answers are out of scope. want. It would help if there were MORE sharks
near the feeding area, not fewer." Many of the
adult dolphins that feed at the beach are
20.A beach has long had a females that nurse their offspring there" is not
dolphin feeding program in relevant to the problem. "Many dolphins that
which fish are given to feed at the beach with their offspring come to
dolphins several times a day; the beach only a few times a month" This
many dolphins get as much as sentence doesn't explain why those dolphins
half of their food each day who take fish near the beach are in danger.
there. Although dolphins that "Adult dolphins that feed at the beach spend
first benefit from the program much less time teaching their offspring how to
as adults are healthy and long- catch fish in the wild than do other adult
lived, their offspring have a dolphins" This is correct. Because of lack of
lower life expectancy than time for teaching off-springs don't know how
offspring of dolphins that feed to catch fish themselves.
exclusively in the wild.Which
of the following, if true, most Directions: In each question three statements
helps to explain the lower life followed by two conclusions numbered I and II
expectancy of offspring of have been given. You have to take the given
dolphins feeding at the beach
statement to be true even if they seem to be at
compared to other young variance with commonly known facts and then
dolphins? decide which of the given conclusions logically
A. Sharks that prey on dolphins are less follows from the given statements. Give
common in the open seas off the answer
beach than in many other areas of the
open seas where dolphins congregate. 21.Statements:
B. Many of the adult dolphins that feed at 0% frocks are pants.
the beach are females that All shorts are frocks.
A few jackets are pants.
C. nurse their offspring there.
Conclusions:
D. Many dolphins that feed at the beach I. Not a single pant is short.
with their offspring come to the beach II. A good number of jackets are not frocks.
only a few times a month.
E. If only conclusion I follows.
E. Adult dolphins that feed at the beach
F. If only conclusion II follows.
spend much less time teaching their
offspring how to catch fish in the wild G. If neither conclusion I nor II follows.
than do other adult dolphins. H. If both conclusion I and II follows.
I. If either conclusion I or II follow
Answer: D
Explanation: Answer: D
Explanation:
Conclusions:
Checking C1: Not a single pant is short. Using
I. At least some jackets are frocks.
S2 and S1, All shorts are frocks + '0% frocks are
II. No jacket is a frock.
pants' or 'No frocks are pants' = No shorts are
pants or No pants are shorts which is same as A. If only conclusion I follows.
'Not a single pant is short'. Hence, C1 follows. B. If only conclusion II follows.
Checking C2: A good number of jackets are not C. If neither conclusion I nor II follows.
frocks. A few jackets are pants + Converse of S1
D. If both conclusion I and II follows.
(which is 'No pants are frocks) = 'A few jackets
are not frocks' which is same as 'A good E. If either conclusion I or II follows.
number of jackets are not frocks'. Hence, C2 Answer: E
follows as well. Clearly, option D is the correct Explanation:
answer. Checking C1: At least some jackets are frocks.
Using S3 and S2, Some jackets are pants + No
22.Statements: pants are frocks = Some jackets are not frocks.
All frocks are pants. Hence, C1 doesn't follow. Checking C2: No
100% shorts are frocks. jacket is a frock. Clearly, we can observe that
Some jackets are pants. we have already got a definite conclusion
above between the classes 'jacket' and 'frock'.
Conclusions: Hence, C2 doesn't follow either. However, we
I. All pants are shorts. closely observe, we can find that both C1 and
II. All shorts being jackets is a possibility. C2 together form a complementary pair and
therefore either C1 or C2 definitely follows.
A) If only conclusion I follows. Option E is hence the correct answer.
B) If only conclusion II follows.
24.Statements:
C) If neither conclusion I nor II follows.
All bottles are jugs.
D) If both conclusion I and II follows. All pans are jugs.
E) If either conclusion I or II follows. Some jugs are not mugs.
Answer: B
Checking C1: All pants are shorts. Using S2 and Conclusions:
S1, we get '100% shorts are frocks' or 'All I. Some bottles are not pans.
shorts are frocks' + 'All frocks are pants' = All II. Some mugs may not be jugs.
shorts are pants. Clearly, C1 doesn't follow. A. If only conclusion I follows.
Checking C2: All shorts being jackets is a
B. If only conclusion II follows.
possibility. As there is no negative statement,
possibility between 'shorts' and 'jackets' does C. If neither conclusion I nor II follows.
exist. C2, hence, follows. Evidently, option B is D. If both conclusion I and II follows.
the correct answer. E. If either conclusion I or II follows.
Answer: B
23.Statement: Explanation:
All the frocks are trousers.
No pants are frocks. Checking Conclusion I: Some bottles are not
Some jackets are pants. pans. Here, neither S1 nor S2 is a negative
statement, a negative conclusion between the I. No tube is bird.
classes of 'Bottles' and 'Pans' is not possible. II. All birds being cubes is a possibility.
C1, hence, doesn't follow. Checking Conclusion A. If only conclusion I follows.
II: Some mugs may not be jugs. In S3 it's given
B. If only conclusion II follows.
that 'Some jugs are not mugs'. Here, we are not
sure of the elements of the class 'Mugs'. C. If neither conclusion I nor II follows.
Clearly, we can say that 'Some mugs may not D. If both conclusion I and II follows.
be jugs'. C2, hence, follows. Option B is hence E. If either conclusion I or II follows.
the correct answer. Answer: B
25.Statements: Explanation:
Some printers are scanners.
Checking Conclusion I : No tube is bird. As we
Some scanners are microphones.
can see that the middle term 'Sky' is not
Many microphones are speakers. distributed even once in either S2 or S3, we
can't define a relationship between the classes
Conclusions:
'tube' and 'bird'. C1, hence, doesn't follow.
I. Some printers are speakers. Checking Conclusion II : All birds being cubes is
II. Not a single printer is a speaker.
a possibility. Since we can't define a
A. If only conclusion I follows. relationship between the classes 'cube' and
B. If only conclusion II follows. 'bird', possibilities between them do follow.
C2, hence, follows here. Option B is hence the
C. If neither conclusion I nor II follows.
correct answer
D. If both conclusion I and II follows.
27.Statements:
E. If either conclusion I or II follows.
No dancers are actors.
Answer: E Some actors are artists.
Explanation: No artist is artisan.
Checking Conclusion I and II together: 'Some
printers are speakers' and 'Not a single printer Conclusions:
is speaker' Clearly, all the statements are I I. Some artists are not dancers.
type, we can't define a relationship between II. Some artisans are not actors.
classes scanner exist in two different A. If only conclusion I follows.
statements. Similarly, we can't define a
B. If only conclusion II follows.
relationship between the classes 'printer' and
'speaker' either. But, C1 is an I type statement C. If neither conclusion I nor II follows.
and C2 an E type, and they together form an D. If both conclusion I and II follows.
E+I combination (Complementary Pair). E. If either conclusion I or II follows.
Clearly, either C1 or C2 follows. Option E is Answer: A
hence the correct answer. Explanation:
26.Statements: Checking Conclusion I : Some artists are not
All tubes are cubes. dancers. Some artists are actors (Converse of
No cube is sky. S2) + No actor is dancer (Converse of S1) =
No bird is sky. Some artists are not dancers. Clearly, C1
follows. Checking Conclusion II : Some artisans
Conclusions: are not actors. Some actors are artists + No
artist is artisan = Some actors are not artisans. B. If only conclusion II follows.
Since converse of an O type statement is not C. If neither conclusion I nor II follows.
possible, C2 doesn't follow. Option A is hence
D. If both conclusion I and II follows.
the correct answer.
E. If either conclusion I or II follows.
28.Statement: Answer: E
All rivers are seas. Explanation:
All lakes are seas. Checking Conclusion I and II together : 'Some
Some seas are not oceans. plates are bowls' and 'Not a single plate is
bowl' Clearly, all the statements are I type, we
Conclusions: can't define a relationship between classes
I. Some rivers are not lakes. that exist in two different statements.
II. Some oceans may not be seas. Similarly, we can't define a relationship
A. If only conclusion I follows. between the classes 'plate' and 'bowl' either.
B. If only conclusion II follows. But, C1 is an I type statement and C2 an E type,
and they together form an E+I combination.
C. If neither conclusion I nor II follows.
Clearly, either C1 or C2 follows. Hence, option
D. If both conclusion I and II follows. E is correct.
E. If either conclusion I or II follows.
Answer: B 30.Statements:
Explanation: All radios are electric goods.
All table-lamps are electric goods.
Checking Conclusion I : Some rivers are not
lakes. Here, neither S1 nor S2 is a negative
Conclusions:
statement, a negative conclusion between the
I. Some radios are table-lamps.
classes of 'rivers' and 'lakes' is not possible. C1,
II. Some table-lamps are radios.
hence, doesn't follow. Checking Conclusion II :
Some oceans may not be seas. In S3 it's given
that 'Some seas are not oceans'. Here, we are A. If only conclusion I follows.
not sure of the elements of the class 'oceans'. B. If only conclusion II follows.
Clearly, we can say that 'Some oceans may not C. If neither conclusion I nor II follows.
be seas'. C2, hence, follows. Option B is hence
D. If both conclusion I and II follows.
the correct answer.
E. If either conclusion I or II follows.
29.Statements: Answer: D
Some plates are knives. Explanation:
Some knives are bottles.
Checking C1 : Some radios are table-lamps. We
Many bottles are bowls.
can observe that the class 'radios' and 'table-
lamps' are present in different statements and
Conclusions:
the middle term 'electric goods' is not being
I. Some plates are bowls.
distributed even once in either of the
II. Not a single plate is bowl.
statements. Therefore, no definite relationship
is possible between these two classes. C1,
A. If only conclusion I follows. doesn't follow. Checking C2 : Some table-lamps
are radios. Following the same logic discussed Answer: E
above, we can state that C2 doesn't follow Explanation:
either. Hence, option D is correct.
Checking C1 : This thief is a man. Using S2 and
S1, we get This thief is a graduate (A) + A
31.Statements: graduate is a man (A) ⇒ This thief is a man.
Hence, C1 follows. Checking C2 : Some men are
No man is a monkey.
thieves. Converse of the conclusion derived
John is a man.
above ⇒ Some men are thieves. Hence, C2
follows as well. Hence, option E is correct.
Conclusions:
I. John is not a monkey.
II. John may or may not be a monkey. 33.Statements:
A. If only conclusion I follows. Only first divisioners are admitted.
B. If only conclusion II follows. Ram is a first divisioner.
35.Statements:
No wire is pin.
Some pins are mugs.
Conclusions:
I. All mugs being wires is a possibility.
II. Some mugs are not wires.
A. If only conclusion I follows.
B. If only conclusion II follows.
C. If neither conclusion I nor II follows.
D. If both conclusion I and II follows.
E. If either conclusion I or II follows.
Answer: B
Explanation:
Answer: